Skip to main content

Section 4.4 Caractérisation d'une mesure et unicité.

Même si les tribus, notamment la tribu des boréliens, sont typiquement moins monstrueuses que \(\mathcal P(X)\) tout entier, leur stabilité par union (et intersection) dénombrable en fait tout de même des familles très riches.

Ainsi, si on a deux mesures \(\mu\) et \(\nu\) sur une tribu \(\mathscr T\text{,}\) que l'on suspecte d'être égales, il est généralement assez fatigant de vérifier que \(\mu(A)=\nu(A)\) pour tout \(A\in \mathscr T\text{.}\)

D'où la question: quel raccourci peut-on prendre pour démontrer l'égalité de deux mesures ?

En algèbre linéaire (en dimension finie), si on dispose d'une famille génératrice de vecteurs \(\{v_1,...,v_m\}\text{,}\) pour vérifier une propriété sur \(\Vect(v_1,...,v_m)\text{,}\) il suffit souvent de montrer qu'elle est vraie pour chacun des \(v_i\text{.}\)

Par analogie, on pourrait imaginer que si on a une tribu \(\T\) engendrée par une famille de sous-ensembles \(\mathcal F\text{,}\) et deux mesures \(\mu_1\) et \(\mu_2\) sur \((X,\T)\) telles que

\begin{equation*} \forall A\in \mathcal F,\quad \mu_1(A)=\mu_2(A) \end{equation*}

alors elles sont égales:

\begin{equation*} \forall A\in \T=\sigma(\mathcal F),\quad \mu_1(A)=\mu_2(A) \end{equation*}

Malheureusement, ce n'est pas si simple:

Exercice 4.4.1. Mesures égales sur un ensemble générateur, mais pas partout.

(a)

Considérons un ensemble tout simple

\begin{equation*} X=\{1,2,3,4\} \end{equation*}

muni de la tribu \(\mathcal P(X)\text{,}\) et la famille de sous-ensembles

\begin{equation*} \mathcal F= \{\{1,2\},\{2,3\}\} \subset \mathcal P(X) \end{equation*}

Montrer que \(\sigma(\mathcal F) = \mathcal P(X)\)

(b)

On définit deux mesures sur \((X,\mathcal P(X))\) par

\begin{gather*} \mu_1(\{1\})=\mu_1(\{2\})=\mu_1(\{3\})=\mu_1(\{4\})=1\\ \mu_2(\{1\})=\mu_2(\{3\})=0;\ \mu_1(\{2\})=\mu_1(\{4\})=2 \end{gather*}

Montrer que

\begin{equation*} \forall A\in \mathcal F,\quad \mu_1(A)=\mu_2(A) \end{equation*}

mais que

\begin{equation*} \exists A\in \mathcal P(X=,\quad \mu_1(A)\neq\mu_2(A) \end{equation*}

(c)

Considérons la tribu des boréliens \(\B(\R)\) sur \(\R\text{.}\) Alors, comme on a déjà vérifié 3.3.4,

\begin{equation*} \B(\R)=\sigma(\underbrace{\{\lbb a,+\infty \rbb, a \in \R\}}_{:=\mathcal F}) \end{equation*}

On considère d'une part \(\mu_1\) la mesure de Borel qu'on a construite à la section précédente 4.3, et d'autre part notons \(\mu_2\) la mesure de comptage 4.2.3 sur \((\R,\B(\R))\) 1 :

\begin{equation*} \mu_2(A)=\begin{cases} \Card(A) \amp\text{ si } A\text{ est fini;}\\ +\infty \amp\text{ sinon.}\end{cases} \end{equation*}

A nouveau, montrer que

\begin{equation*} \forall A\in \mathcal F,\quad \mu_1(A)=\mu_2(A) \end{equation*}

mais que

\begin{equation*} \exists A\in \mathcal P(X=,\quad \mu_1(A)\neq\mu_2(A) \end{equation*}

On ne va donc pas pouvoir se contenter de n'importe quelle famille génératrice: il va falloir quelques propriétés supplémentaires.

Subsection 4.4.1 Les \(\lambda\)-systèmes et les \(\pi\)-systèmes.

Il se trouve que pour les questions d'unicité, les familles génératrices de tribus ne suffisent pas. On introduit donc ici un peu de vocabulaire supplémentaire:

Définition 4.4.1. Pi-systèmes.

Une famille de sous-ensembles \(P \subset \mathcal P(X)\) est un \(\pi\)-système si elle est stable par intersection finie:

\begin{equation*} \forall A_1,\ldots,A_m\in P,\ A_1\cap....\cap A_m\in \end{equation*}

Exercice 4.4.2. Exemples.

(a)

Notons \(X=\{1,2,3,4\}\) et \(\mathcal F= \{\{1,2\},\{2,3\}\}\text{.}\) Est-ce que \(\mathcal F\) est un \(pi\)-système ?

(b)

Si \(E\) est un e.v.n, est ce que l'ensemble \(\mathcal O_E\) des ouverts de \(E\) est un \(\pi\)-système ? Et l'ensemble des fermés ?

(c)

Montrer que \(\{\lbb a, b\lbb, a,b \in\R, a\lt b\}\) et \(\{\lbb a,+\infty \rbb, a \in \R\}\) sont des \(\pi\)-systèmes.

Il va nous falloir une autre lettre grecque 2 :

Définition 4.4.2. Lambda-système/classe monotone.

Une famille de sous-ensembles \(L \subset \mathcal P(X)\) est un \(\lambda\)-système, ou classe monotone si

  • \(X \in L\text{;}\)

  • \(L\) est stable par différence: Si \(A,B \in L\text{,}\) \(A \subset B\) alors \(B \setminus A \in L\text{;}\)

  • \(L\) est stable par union dénombrable croissante: Si \((A_n)_n\) est telle que \(A_n \in L\) et \(A_n \subset A_{n+1}\) (autrement dit \((A_n)_n\) est une suite croissante) alors \(\bigcup A_n \in L\text{.}\)

Exercice 4.4.3. Exemples.

(a)

Justifier que toute tribu est une classe monotone.

On considère l'ensemble \(X=\{1,2,3,4\}\) et

\begin{equation*} \mathcal L=\{\emptyset, X,\{1,2\},\{1,3\},\{1,4\},\{2,3\},\{2,4\},\{3,4\}\} \end{equation*}

Montrer que \(\mathcal L\) est une classe monotone, mais pas une tribu de parties de \(X\text{.}\)

(b)

Les familles \(\{\lbb a, b\lbb, a,b \in\R, a\lt b\}\) et \(\{\lbb a,+\infty \rbb, a \in \R\}\) sont-elles \(\lambda\)-systèmes ?

(c)

Soient \(\Pp_1,\Pp_2\) deux mesures de probabilité sur un ensemble mesurable \((X,\T)\text{.}\) Montrer que

\begin{equation*} \mathcal L=\{A\in\T, \Pp_1(A)=\Pp_2(A)\} \end{equation*}

est un \(\lambda\)-système.

Remarque 4.4.3.

Pourquoi toutes ces lettres grecques ? Dynkin 3  seul le sait, mais la légende prétend que \(\pi\) voudrait ici dire "produit", car dans les vieux livres de théorie de la mesure et probabilité, l'intersection \(A\capB\) est souvent notée \(AB\text{;}\) et \(\lmabda\) signifierait "limite", en raison des suites croissantes \((A_n)_n\) qu'on considère, et la propriété des mesures 4.1.7 qui nous dit que, dans ce cas,

\begin{equation*} \mu\left(\bigcup_n A_n \right) = \lim_{n\rightarrow\infty}\mu(A_n) \end{equation*}

Mais d'autres disent que, dans le tas de lettres grecques \(\mu, \nu\) étaient déjà prises pour désigner des mesures, \(\sigma\) aussi pour parler de \(\sigma\)-additivité, et que \(\lambda\) et \(\pi\) étaient dispos.

Comme les tribus, on peut définir le \(\lambda\)-système engendré par une famille donnée:

Le lien entre \(\lambda\)-systèmes et tribus est donné par la propriété suivante:

Vérifions les trois propriétés des tribus.

  1. Puisque \(\Lambda\) est stable par différence, \(\emptyset=X\setminus X\in\Lambda\text{.}\)

  2. Puisque \(X\in \Lambda\text{,}\) et puisque \(\Lambda\) est stable par différence, en particulier \(\Lambda\) est stable par complémentaire: pour tout \(A\in \Lambda\text{,}\) \(A^c=X\setminus A\in \Lambda\text{.}\)

  3. Montrons maintenant que \(\Lambda\) est stable par union dénombrable. Soit \((A_n)_n\in \Lambda^\mathbb N\) une famille dénombrable d'éléments de \(\Lambda\text{.}\) Pour tout \(n\in \mathbb N\text{,}\) posons

    \begin{equation*} B_n=\bigcup_{k=0}^n A_k \end{equation*}

    Alors \(B_n^c= \bigcap_{k=0}^n A_k^c\in \Lambda\text{,}\) puisque \(\Lambda\) est stable par passage au complémentaire et par intersection finie. Donc \(B_n=(B_n^c)^c \in \Lambda\text{.}\) De plus, \(B_n\subset B_{n+1}\) pour tout \(n\in \mathbb N\text{,}\) donc

    \begin{equation*} \bigcup_{n\geq 0}A_n=\bigcup_{n\geq 0}B_n \in \Lambda. \end{equation*}

En utilisant ceci, on peut montrer que le \(\lambda\)-système \(\lambda(\mathscr F)\) et la tribu \(\sigma(\mathscr F)\) engendrés par certaines familles \(\mathscr F\) coïncident:

On procède par double inclusion.

\(\boxed{\subset}\) Remarquons que toute tribu est un \(\lambda\)-système, donc une tribu qui contient \(\mathscr F\) est en particulier un \(\lambda\)-système qui contient \(\mathscr F\text{,}\) donc:

\begin{equation*} \lambda(\mathscr F)=\bigcap_{\Lambda\ \lambda-\text{système, } \mathscr F \subset \Lambda} \Lambda \subset \bigcap_{\mathscr T \text{tribu, } \mathscr F \subset \mathscr T} \mathscr T = \sigma(\mathscr F) \end{equation*}

\(\boxed{\supset}\) Reste à montrer l'inclusion réciproque. Par définition de \(\sigma(\mathscr F)\text{,}\) il suffit de montrer que \(\lambda(\mathscr F)\) est une tribu qui contient \(\mathscr F\text{.}\) Or, d'après la proposition précédente, il suffit pour cela d'établir que \(\lambda(\mathscr F)\) est stable par intersection finie.

Soit \(E\in \mathscr F\text{,}\) on considère la sous-famille des éléments de \(\Lambda\) sont l'intersection avec \(E\) est encore dans \(\Lambda\text{:}\)

\begin{equation*} \Lambda_E=\{A\in \mathscr \Lambda(\mathscr F), A\cap E \in \Lambda(\mathscr F)\} \end{equation*}

Alors \(\Lambda_E\) est un \(\lambda\)-système qui contient \(\mathscr F\text{:}\)

  1. \(X\cap E= E \in \Lambda(\mathscr F)\text{,}\) donc \(X \in \Lambda_E\text{.}\)

  2. Si \(A,B\) sont dans \(\Lambda_E\text{,}\) avec \(A \subset B\text{,}\) alors \((B\setminus A)\cap E=(B\cap E)\setminus(A \cap E)\) est la différence de deux éléments de \(\Lambda(\mathscr F)\text{,}\) donc \((B\setminus A)\cap E\in \Lambda(\mathscr F)\text{,}\) donc \(B\setminus A \in \Lambda_E\text{.}\)

  3. Soit \((A_n)_n\) une suite croissante d'éléments de \(\Lambda_E\text{.}\) Alors, puisque \(\Lambda(\mathscr F)\) est stable par union croissante,

    \begin{equation*} E\cap \left(\bigcup_n A_n\right)= \bigcup_n (E\cap A_n)\in \Lambda(\mathscr F) \end{equation*}

    donc l'union des \(A_n\) est bien dans \(\Lambda_E\text{.}\)

  4. Enfin, puisque \(\mathscr F\) est stable par intersection finie, on a, pour tout \(F\in \mathscr F\text{,}\) \(F\cap E \in \mathscr F \subset \Lambda(\mathscr F)\text{,}\) donc \(F\in \Lambda_E\text{.}\)

On en déduit que \(\Lambda_E= \Lambda(\mathscr F)\text{,}\) autrement dit

\begin{equation*} \forall E \in \mathscr F,\ \forall A \in \Lambda(\mathscr F), A\cap E \in \Lambda(\mathscr F). \end{equation*}

En utilisant ceci, on montre de même que, pour tout \(B\in \Lambda(\mathscr F)\text{,}\) la famille

\begin{equation*} \Lambda_B=\{A\in \mathscr \Lambda(\mathscr F), A\cap B \in \Lambda(\mathscr F)\} \end{equation*}

est un \(\lambda\)-système qui contient \(\mathscr F\text{,}\) donc \(\Lambda_B=\Lambda(\mathscr F)\text{.}\) Autrement dit,

\begin{equation*} \forall A \in \Lambda(\mathscr F),\ \forall B \in \Lambda(\mathscr F), A\cap B \in \Lambda(\mathscr F). \end{equation*}

donc \(\Lambda(\mathscr F)\) est stable par intersection finie. C'est donc une tribu, qui de plus contient \(\mathscr F\text{,}\) d'où \(\sigma(\mathscr F)\subset \Lambda(\mathscr F)\text{.}\)

Subsection 4.4.2 Application : Caractérisation des mesures

En utilisant les \(\lambda\)-systèmes, on montre le critère d'égalité suivant dans le cas des mesures finies:

Supposons d'abord que \(\mu\) et \(\nu\) sont deux mesures finies. Considérons la famille

\begin{equation*} L = \{A\in \mathscr T,\lambda(A)=\mu(A)\} \end{equation*}

Alors, par hypothèse, \(\mathscr F\subset L\text{.}\) De plus, \(L\) est un \(\lambda\)-système:

  1. Puisque \(X\in \mathscr F\text{,}\) on a \(\mu(X)=\nu(X)\text{,}\) donc \(X\in L\text{.}\)

  2. Soient \(A,B\) deux éléments de \(L\) tels que \(A\subset B\text{.}\) Alors \(\mu(A)=\nu(A) \lt \infty\text{,}\) \(\mu(B)=\nu(B)\lt \infty\text{,}\) puisque \(\mu\) et \(\nu\) sont supposées finies. On a donc

    \begin{equation*} \mu(B\setminus A)= \mu(B)-\mu(A)=\nu(B)-\nu(A)=\nu(B\setminus A), \end{equation*}

    donc \(B\setminus A \in L\text{.}\)

  3. Soit \((A_n)_n\) une suite croissante d'éléments de \(L\text{.}\) Alors

    \begin{equation*} \mu\left(\bigcup_n A_n\right)= \lim_{n\rightarrow\infty} \mu(A_n)= \lim_{n\rightarrow\infty} \nu(A_n) = \nu\left(\bigcup_n A_n\right) \end{equation*}

    donc \(\bigcup_n A_n \in L\text{.}\)

On en déduit que \(\lambda(\mathscr F)\subset L\text{.}\) De plus, par la proposition Proposition 4.4.6, on a \(\lambda(\mathscr F)=\sigma(\mathscr F)=\mathscr T\text{,}\) donc \(\mathscr T \subset L\text{:}\) pour tout \(A\in \mathscr T\text{,}\) \(\mu(A)=\nu(A)\text{.}\)

On peut en fait relâcher un peu l'hypothèse de finitude des mesures:

Pour \(n\in \mathbb N\text{,}\) on définit deux mesures finies \(\mu_n\) et \(\nu_n\) sur \((X,\mathscr T)\) par \(\mu_n: A\in \mathscr T \mapsto \mu(A\cap E_n)\) et \(\nu_n:A\in\mathscr T\mapsto \nu(A\cap E_n)\text{.}\)

Alors, pour tout \(A\in \mathscr F\text{,}\) \(\mu_n(A)=\nu_n(A)\text{.}\) Puisque la famille \(\mathscr F\) vérifie les hypothèses du corollaire précédent, on en déduit que \(\mu_n=\nu_n\) pour tout \(n\in\mathbb N\text{.}\)

Or, pour tout \(A\in T\text{,}\) la propriété sur les unions croissantes (Proposition 4.1.7) donne

\begin{equation*} \mu(A)=\lim_{n\rightarrow\infty}\mu(A\cap E_n)=\lim_{n\rightarrow\infty}\mu_n(A)=\lim_{n\rightarrow\infty} \nu_n(A)=\lim_{n\rightarrow\infty} \nu(A\cap E_n)=\nu(A), \end{equation*}

donc on a bien \(\mu=\nu\text{.}\)

Remarque 4.4.9.

La stratégie de cette preuve, qui consiste à utiliser une suite croissante d'ensembles pour se ramener à des mesures finies, est très courante, d'autant plus que la plupart des mesures "usuelles" sont \(\sigma\)-finies.

Armé de ce résultat, on peut enfin démontrer que la mesure de Borel qu'on a construite à la section précédente 4.3 est bien la seule et unique mesure qui mesure correctement les intervalles:

Notons \(\mathscr I\) la famille de tous les intervalles de \(\mathbb R\text{.}\) Alors

  1. \(\mathscr I\subset \mathscr B(\mathbb R)\text{;}\)

  2. \(\mathbb R\in \mathscr I\) et \(\mathscr I\) est stable par intersection finie;

  3. La famille \(\mathscr I\) engendre \(\mathscr B(\mathbb R)\text{:}\) \(\sigma(\mathscr I)=\mathscr B(\mathbb R)\text{.}\)

Supposons maintenant qu'il existe deux mesures \(\mu\) et \(\mu'\) sur \((\mathbb R,\mathscr B(\mathbb R))\text{,}\) invariantes par translation et généralisant la longueur des intervalles: pour tout \(I\in \mathscr I, \mu(I)=\ell(I)=\mu'(I)\text{.}\)

Pour \(n\in\mathbb N\text{,}\) posons \(I_n=\rbb -n,n\lbb \in \mathscr I\text{.}\) Alors, pour tout \(n\in\mathbb N\) \(I_n\subset I_{n+1}\text{,}\) \(\mu(I_n)=\mu'(I_n)=2n\lt \infty\) et \(\mathbb R= \bigcup_n I_n\text{.}\)

On peut donc appliquer la corollaire Corollaire 4.4.8, ce qui donne \(\mu=\mu'\text{.}\)

La mesure de comptage est en fait définie sur le monstrueux \(\P(\R)\text{,}\) mais on la restreint ici aux boréliens.
Il est recommandé pour ce cours d'aimer la moussaka et le gyros. Sinon, voir ici: https://fr.wikipedia.org/wiki/Alphabet_grec
en.wikipedia.org/wiki/Dynkin_system